ORTHOPEDIC MCQS ONLINE 013 SPORT

ORTHOPEDIC MCQS ONLINE 013 SPORTS 

2013 Sports Medicine Self-Assessment Examination by Dr.Dhahirortho

 

 

1

 

 

 

 

 

Question 1Figure 1 is the MRI scan of a 19-year-old man who has an acute anterior shoulder dislocation. The bony fragment occupies 10% of the glenoid articular surface. What is the most appropriate treatment?

 

  1. Open structural iliac crest graft

  2. Open reduction and internal fixation

  3. Arthroscopic coracoid transfer

  4. Arthroscopic repair incorporating the bone lesion

 

DISCUSSION --The MRI scan shows a bony Bankart lesion involving less than 20% of the glenoid joint surface. A recent

series reported high success rates after arthroscopic treatment when the defect is incorporated into the repair. Anterior bony deficiencies occupying more than 25% to 30% of the glenoid joint surface treated with soft-tissue repair only are associated with high recurrence rates. In these patients, an open or arthroscopic coracoid transfer or structural iliac crest graft should be considered. Open reduction and internal fixation has been reported for treatment of large acute glenoid rim fractures but is not recommended for recurrent anterior shoulder instability in the setting of a 10% glenoid rim fracture. PREFERRED RESPONSE: 4

 

Question 2--A 19-year-old running back lands directly on his anterior knee after being tackled. He has mild anterior knee pain, a trace effusion, a 2+ posterior drawer, a grade 1+ stable Lachman, no valgus laxity, and negative dial tests at 30 degrees and 90 degrees. What is the best treatment strategy at this time?

 

  1. Physical therapy with a focus on quadriceps strengthening

  2. Physical therapy and delayed posterior cruciate ligament (PCL) reconstruction

  3. PCL reconstruction

  4. PCL and posterolateral corner reconstruction

 

DISCUSSION-This patient has likely sustained an isolated PCL injury. The examination is consistent with a grade II injury to the PCL. In this scenario, the best initial option is nonsurgical treatment and return to play as symptoms subside and strength improves. Physical therapy with a focus on quadriceps strengthening and delayed PCL reconstruction is not the answer because this patient can likely be treated without surgery.The absence of valgus laxity and negative dial testing findings suggest that an injury to the posteromedial and posterolateral corners has not occurred. Initial nonsurgical treatment is indicated for this patient. If he completes rehabilitation and experiences persistent disability with anterior and/or medial knee discomfort or senses the knee is “loose,” PCL reconstruction should be considered at that time. PREFERRED RESPONSE: 1

 

Question 3-Figure 3 is the clinical photograph of a 20-year-old college soccer player who has a 7-day history of worsening left ankle pain and swelling after being slide-tackled in a game.

Radiograph findings of his ankle and foot are normal. He complains of malaise. His history includes a severe ankle sprain 3 months ago. The sprain caused him to miss half the season, but he was able to play in the last 2 games. What is the most appropriate treatment?

 

  1. Incision and drainage

     

     

     

  2. Ice the ankle but don't let him play.

  3. Topical antibiotics for 7 days with an occlusive dressing

  4. Debridement in the training room followed by 5 days of oral antibiotics

 

DISCUSSION--The clinical photograph shows a skin infection with an appearance consistent with methicillin-resistant Staphylococcus aureus.

This infection should be clinically incised and allowed to drain and a

course of antibiotics should follow. If this infection is not promptly treated with debridement, it likely will worsen and potentially spread to other teammates. Antibiotics are secondary to surgical debridement but are a necessary adjunct. Although this patient has a history of severe sprain, his malaise and skin appearance do not correlate with a ligament injury or fracture. Debridement in the training room is not appropriate and would likely not fully decompress the fluid collection. PREFERRED RESPONSE: 1

 

RESPONSES FOR QUESTIONS 4 AND 5

  1. Semimembranosis tendonitis

  2. Patellar tendonitis

  3. Iliotibial band friction syndrome

  4. Quadriceps tendonitis

 

Please select the most likely diagnosis listed above for each clinical situation.

 

Question 4-A 23-year-old otherwise healthy 6-ft, 4-in basketball player complains of pain in his knees. An examination reveals localized tenderness to palpation over the inferior pole of the patella. The patient notes a significant exacerbation of his pain when the examiner takes the knee from flexion to extension.

 

PREFERRED RESPONSE: 2

 

Question 5-A 22-year-old 6-ft, 2-in Olympic cyclist has had knee pain for 2 months. Examination reveals localized tenderness to palpation over the lateral femoral epicondyle most notably at 30 degrees of flexion.

 

PREFERRED RESPONSE: 3

DISCUSSION FOR QUESTIONS 4 AND 5

Patellar tendonitis is common in jumping sports such as basketball and volleyball. The pain is localized to the inferior border of the patella and is exacerbated by extension of the knee. Treatment for the vast majority of patients is nonsurgical and includes nonsteroidal anti-inflammatory drugs, physical therapy,and orthoses (patella tendon strap). Iliotibial band friction most commonly occurs in cyclists and runners (especially those who run up hills) and is a result of abrasion between the iliotibial band and the lateral femoral condyle. Localized tenderness with the knee flexed at 30 degrees is common. The Ober test may be helpful in making the diagnosis. Semimembranosis tendonitis most commonly occurs in male athletes during their fourth decade of life. The diagnosis is usually made with an MRI scan or nuclear imaging. Quadriceps tendonitis is similar to patellar tendonitis but is much less common. The pain may be associated with clicking and is localized to the superior border of the patella.

 

Question 6-A 26-year-old weightlifter had increasing pain in his left shoulder for 4 months. Nonsurgical treatment consisting of anti-inflammatory medication, corticosteroid injections, and rest failed to alleviate his symptoms. He underwent an arthroscopic distal clavicle resection with excision of the distal 8 mm of clavicle (Mumford procedure). Three months after surgery, he reported popping by his clavicle and mild pain. His clavicle demonstrated mild posterior instability on examination without any obvious deformity on his radiographs. What structures were compromised during his excision?

 

  1. Anterior and superior acromioclavicular joint ligaments

  2. Posterior and superior acromioclavicular joint ligaments

  3. Conoid ligament

  4. Trapezoid ligament

 

DISCUSSIO-The posterior and superior acromioclavicular ligaments provide the most restraint to posterior translation of the acromioclavicular joint and must be preserved during a Mumford procedure. Anterior and superior acromioclavicular joint ligaments are the opposite of the preferred response and prevent anterior translation of the clavicle. Injuries to the conoid and trapezoid ligaments are more pronounced with grade III or higher acromioclavicular separations, with superior migration of the clavicle relative to the acromion. PREFERRED RESPONSE: 2

 

CLINICAL SITUATION FOR QUESTIONS 7 THROUGH 9

Question 7-A 19-year-old female field hockey player has a right ankle injury that occurred last night during a game.The patient is on crutches and states that she has not been able to put any weight on her right ankle since the injury. She was running alongside with another player when her right ankle “gave out” and she twisted it, falling to the ground. Physical examination revealed discoloration similar to a hematoma and significant swelling around the lateral ankle area. Pain was elicited during palpation of the anterior talofibular ligament. What examination test should be performed to aid in this diagnosis?

 

  1. Thompson test

  2. External rotation stress test

  3. Anterior drawer test

  4. Squeeze test

PREFERRED RESPONSE: 3

Question 8-Radiographs of the player’s right ankle confirm there are no fractures. With a lateral talar tilt test result of 19 degrees, which additional structure is most likely damaged?

 

  1. Deltoid ligament

  2. Calcaneofibular ligament

  3. Anterior tibiofibular ligament

  4. Posterior tibiofibular ligament

PREFERRED RESPONSE: 2

 

Question 9-What is the most appropriate course of action for this patient’s condition?

  1. Early mobilization and a guided proprioceptive and strengthening rehabilitation program

  2. Extended immobilization in a cast

  3. Surgical intervention

  4. Weight bearing as tolerated in an ankle brace for 6 weeks

PREFERRED RESPONSE: 1

 

DISCUSSION FOR QUESTIONS 7 THROUGH 9

The anterior drawer test is performed with the ankle in 10 degrees of plantar flexion, which results in the greatest amount of translation. The test investigates the integrity of the anterior talofibular ligament with a key distance of translation being 8 to 10 mm. While the patient is sitting and has her knees flexed over the edge of a table or bench, the physician or examiner uses one hand to stabilize the distal leg and with the other applies an anterior force to the heel in an attempt to gap the talus anteriorly from under the tibia. The anterior talofibular ligament and calcaneofibular ligament are both compromised based on the examination findings. The anterior drawer test result reflects injury to the anterior talofibular ligament and a possible injury to the calcaneofibular ligament. A lateral talar tilt test angle measurement greater than 15 degrees reflects a rupture of both anterior talofibular ligament and calcaneofibular ligaments.The diagnosis is a severe lateral ligament complex sprain. Considering the involvement of the anterior talofibular ligament and calcaneofibular ligaments, early mobilization with a cast or controlled ankle movement walker boot has been documented to result in better patient outcomes than compression or air casting.

 

Question 10-A 17-year-old basketball player has a soft-tissue abscess over the anterior aspect of his left knee. The team physician prescribes amoxicillin and the infection resolves. The next week the patient develops fevers and significantly increased pain at the site of the previous infection. What is the most likely diagnosis?

  1. Community-acquired methicillin-resistant Staphylococcus aureus (CA-MRSA)

  2. Tinea corporis

  3. Herpes simplex virus

  4. Group A Streptococcus

 

DISCUSSION--Skin and soft-tissue abscesses should be drained and cultured by the treating physician whenever possible.Antibiotic therapy should be guided by antibiotic sensitivities derived from the cultures to identify cases of CA-MRSA and prevent severe recurrent infections. These infections have been associated with significant morbidity, with up to 70% of players requiring hospitalization. A high index of suspicion in at-risk populations is necessary, and empiric treatment with an antibiotic effective against MRSA should be considered until sensitivity results are available. Tinea corporis is a general term for a cutaneous fungal infection. The lesion appears as a well-demarcated erythematous plaque with a raised border and central hypopigmentation, giving it a ring-like appearance. Primary infection with herpes simplex virus can produce constitutional symptoms with burning, tingling, or stinging at the site. Grouped vesicles with clear fluid 1 mm to 2 mm in size form on an erythematous base and then rupture, leaving moist ulcers or crusted plaques. Amoxicillin is appropriate empiric antibiotic therapy for group A Streptococcus, so a recurrent infection is less likely with this organism. PREFERRED RESPONSE: 1

 

Question 11-Figure 11 is the anteroposterior radiograph of a 20-year-old dancer who fell during his routine and injured his right foot. What is the most appropriate treatment?

 

 

  1. Closed reduction and cast

  2. Open reduction and internal fixation

  3. In situ percutaneous pinning

  4. Posterior splint immobilization and controlled ankle motion (CAM) walker ambulation

 

DISCUSSION--This patient has sustained a Lisfranc fracture dislocation of the forefoot. To fully restore foot function, an open reduction and internal fixation should be performed to anatomically reduce this dislocation. Closed reduction is unlikely to restore normal foot biomechanics and would likely result in delayed

 

arthritis and joint incongruity. Posterior splint immobilization, CAM walker ambulation, and in situ percutaneous pinning will not adequately reduce the fracture and restore normal function to the foot. PREFERRED RESPONSE: 2

 

Question 12What factor highly correlates with poor outcomes after surgery for femoroacetabular impingement?

  1. Age younger than 20

  2. Degenerative arthritis

  3. Prominence of the femoral head in cam impingement

  4. The patient is a professional athlete

 

DISCUSSION-A systematic review of case studies looking at the results of surgical treatment for femoroacetabular impingement showed good results for most patients, with the exception of those with preoperative radiographs showing osteoarthritis or Outerbridge grade III or grade IV cartilage damage noted intraoperatively. Both Byrd and Jones and Philippon and associates have shown good surgical results for this condition among professional athletes. Likewise, Fabricant and associates demonstrated good surgical results among adolescent patients with an average age of 17.6 years.

PREFERRED RESPONSE: 2

 

CLINICAL SITUATION FOR QUESTIONS 13 THROUGH 16

A 40-year-old man who is a manual laborer has had 3 years of worsening medial-sided left knee pain that has inhibited his ability to work. He reports undergoing a left subtotal medial menisectomy 10 years ago. He has been treated with nonsteroidal anti-inflammatory drugs and 2 different corticosteroids, with the most recent injection given 1 month ago. Each injection provided him with a few weeks of pain control.His medical history is unremarkable and he has smoked 20 cigarettes per day for the last 15 years. His body mass index (BMI) is 22. On examination, he has varus alignment of the involved leg and medial joint line tenderness and no lateral or patellofemoral pain. His knee range of motion is 3 degrees shy of full extension to 130 degrees of flexion. He has negative Lachman and posterior drawer test results. He demonstrates no lateral thrust with ambulation.

 

Question 13-What imaging study is most appropriate to determine treatment options for this patient?

  1. Full-length weight-bearing radiographs of both legs

  2. MRI scan of the left knee

  3. CT scan of the left knee

  4. Ultrasound of the left leg

 

Question 14-What is the most appropriate next step in treatment?

  1. Repeat corticosteroid injection

  2. Trial of a medial unloader brace

  3. MRI scan of the knee to evaluate for recurrent medial meniscus tear

  4. Referral to pain management

PREFERRED RESPONSE: 1

 

PREFERRED RESPONSE: 2

Question 15--The patient is provided with a medial unloader brace that provides substantial pain relief and he is able to work while wearing the brace. After 4 months he returns to work and says that while the brace enable him to work, it is uncomfortable. Consequently, his symptoms return when he is not wearing the brace and he is requesting a surgical intervention for his problem. What is the most appropriate surgical treatment?

  1. Valgus-producing high tibial osteotomy (VPHTO)

  2. Repeat knee arthroscopy

     

  3. Total knee arthroplasty (TKA)

  4. Medial meniscus transplant

PREFERRED RESPONSE: 1

Question 16--The patient is offered a VPHTO. What aspect of his history will determine the most appropriate VPHTO technique?

  1. Prior arthroscopy

  2. Current smoking history

  3. BMI of 22

  4. Age of 40

PREFERRED RESPONSE: 2

DISCUSSION FOR QUESTIONS 13 THROUGH 16

This patient has a classic presentation of postmeniscectomy medial compartment arthritis. The appropriate diagnostic study is weight-bearing radiographs to confirm the diagnosis. An MRI scan will reveal medial compartment arthritis but will not provide information about alignment. A CT scan would be appropriate to detect an occult fracture; however, this condition is not suspected in this clinical scenario. An ultrasound can provide information about fluid collection around the knee or a deep vein thrombosis; however, these conditions also are not suspected in this clinical scenario.

Because the patient has a correctable deformity (gaps 3 mm with valgus stress) and his symptoms are localized to the involved compartment, a trial of a medial unloader brace is appropriate both diagnostically and therapeutically. If unloading the medial compartment resolves the patient’s symptoms, he would be an excellent candidate for an osteotomy. An MRI scan may be obtained to evaluate ligamentous integrity or to evaluate degenerative involvement of the lateral and patellofemoral compartment for presurgicalplanning of an osteotomy; however, the integrity of the medial meniscus has no clinical importance in a patient with severe medial compartment arthritis. A repeat corticosteroid injection is not indicated within 1 month of his last injection, and referral to pain management is not appropriate with other options available to help this patient.A VPHTO is the appropriate intervention considering the patient’s young age, high-functional occupation,examination, radiographic findings, and response to medial unloader bracing. A revision knee arthroscopy would be appropriate for a recurrent medial meniscus tear, but not in a patient with severe medial compartment arthritis. The patient’s young age and high functional requirements are contraindications to TKA. The presence of severe arthritis is a contraindication to medial meniscus transplant. The patient is a candidate for a VPHTO. The technical options include a medial opening-wedge or a lateral closing-wedge osteotomy. Both techniques have advantages and disadvantages; however, a medial opening-wedge osteotomy is contraindicated in a smoker because of concern for nonunion. As a result,current smoking history is the only factor listed that would influence the technique used. The history of prior arthroscopy has no relevance in the decision about which type of osteotomy is appropriate. Normal BMI is between 18.5 and 24.9, so this patient’s BMI is considered normal and would not affect the surgical technique (if this patient were obese, a lateral closing-wedge osteotomy would be considered, but this is controversial). His age of 40 is an indication for HTO but does not influence technique.

 

Question 17-When reconstructing the anterior cruciate ligament (ACL), what is the most common source of potential autograft failure?

  1. Graft choice

  2. Tunnel position

  3. Tibial fixation

  4. Femoral fixation

 

DISCUSSION--Technical failure is the most common reason for ACL reconstruction failure. Tunnel position is the most frequent cause for technical failure. Malpositioning of the tunnel affects the length of the graft, causing either decreased range of motion or increased graft laxity. Although graft choice

 

is an important factor when planning an ACL reconstruction, overall outcomes with autograft tissues are fairly similar. Fixation of the graft at the femoral or tibial end is not as important as tunnel position. PREFERRED RESPONSE: 2

CLINICAL SITUATION FOR QUESTIONS 18 THROUGH 20

A 25-year-old healthy woman injured her left knee while playing professional soccer. She has never injured this knee before. Examination 2 days after the injury occurred reveals the following: a moderate effusion, a positive Lachman test result, and mild lateral tenderness. Range of motion is between 20 degrees and 70 degrees. Radiographs reveal no fracture. An MRI scan reveals a complete rupture of the anterior cruciate ligament (ACL), an effusion, and bone bruises of the lateral femoral condyle and lateral tibial plateau. No meniscal tear is seen. The patient would like to continue playing at the professional level.

Question 18--What is the next treatment step?

  1. Immobilization of the knee for 6 weeks, followed by rehabilitation and delayed ACL reconstruction

  2. Immediate ACL reconstruction

  3. Immediate rehabilitation for 6 months followed by ACL reconstruction if the patient is unstable in a brace

  4. Immediate rehabilitation with delayed ACL reconstruction (when the athlete obtains full knee range of motion)

PREFERRED RESPONSE: 4

Question 19-What is this patient’s risk for developing osteoarthritis (OA) of the knee?

  1. There is no risk for development of knee OA after reconstruction of the ligament.

  2. There is no risk for development of knee OA after a double-bundle ACL reconstruction.

  3. There is no evidence that ACL reconstruction reduces the incidence of knee OA.

  4. There is 100% likelihood that she will develop knee OA after single-bundle ACL reconstruction.

PREFERRED RESPONSE: 3

Question 20-The patient asks if something about her anatomy has resulted in this injury. ACL anatomy differs between men and women in what manner?

  1. There is no significant difference in ACL anatomy between men and women.

  2. A woman’s ACL has a smaller cross-sectional area.

  3. The cross-sectional area of a woman’s ACL is larger.

  4. The intercondylar notch is wider in women than in men.

PREFERRED RESPONSE: 2

DISCUSSION FOR QUESTIONS 18 THROUGH 20

This patient has the clinical findings of an ACL rupture that is confirmed on MRI scan. She is a professional athlete and would like to return to her sport. Immediate ACL reconstruction in the setting of a knee with limited motion carries an increased risk for postsurgical stiffness. Delayed surgery after the patient regains range of motion is the preferred response. It has been shown that a woman’s ACL is smaller in the cross-sectional area.

 

 

 

Question 21-Figure 21 is the radiograph of a 31-year-old man who had left shoulder pain after a fall during a snow boarding jump. Residual displacement of 5 mm after closed reduction is most likely to result in which of the following?

  1. Nonunion

  2. Osteonecrosis

  3. Altered rotator cuff mechanics

  4. Normal shoulder function

 

DISCUSSION--Humerus fractures account for 11% of all fractures among snowboarders and are the second-mostcommon upper-extremity fracture after radius fractures (48%). Surgical fixation is recommended for fractures with residual displacement greater than 5 mm, or 3 mm in active patients involved in frequent overhead activity. Malunion can result in a mechanical block to shoulder abduction or external rotation and altered rotator cuff mechanics, causing weakness. A rich arterial network provides a favorable healing environment for greater tuberosity fractures. Consequently, nonunion and osteonecrosis are uncommon. PREFERRED RESPONSE: 3

 

Question 22-What strategy has proven most effective in preventing transmission of methicillin-resistant Staphylococcus aureus among teammates?

  1. Separate players with infections in a separate locker room or changing area.

  2. Treat teammates of the infected player with prophylactic antibiotics.

  3. Cover any skin lesions with occlusive dressing during sporting activity.

  4. Ban players with infections from any team event.

 

DISCUSSION-Prevention is the key to controlling infections among athletes. Proper hygiene is critical and should mandate showering, hand washing, wearing breathable clothing, and shower sandals. The sharing of towels or athletic equipment should be forbidden. Daily skin surveillance by athletes, trainers, and physicians can allow early recognition and treatment initiation during the early stages of infection,limiting risk for further transmission. Additionally, disinfecting shared equipment, covering lesions with occlusive dressing during sporting activity, and restricting the contact activities of infected athletes can limit risk for an infectious outbreak among teammates. PREFERRED RESPONSE: 3

 

 

 

Question 23 -Figure 23 is the T2 axial MRI scan of a 21-year-old man who was injured while playing for his college football team. His pain was aggravated with blocking maneuvers and alleviated with rest, and he had to stop playing because of the pain. What examination maneuver most likely will reproduce his pain?

  1. Forward elevation in the scapular plane

  2. External rotation and abduction

  3. Flexion, adduction, and internal rotation

  4. Flexion and abduction

 

DISCUSSION -This patient has a mechanism of injury and MRI consistent with a posterior labral tear and posterior instability.

Flexion, adduction, and internal rotation produce a net posterior

vector on the glenohumeral joint and should reproduce this patient's symptoms. Pain or instability with the arm elevated in the scapular plane describes an impingement sign. Pain or instability with the arm in external rotation and abduction describes the apprehension sign. Pain or instability with the arm in flexion and abduction is a nonspecific finding. PREFERRED RESPONSE: 3

CLINICAL SITUATION FOR QUESTIONS 24 AND 25

During the third quarter of a high school football game, a 16-year-old running back gets tackled and limps off the field. During the initial sideline evaluation, he has tenderness on the right iliac crest. He is a little dizzy, has a headache, and tells you, “I need to get back in the game to help the team score before halftime.”

Question 24-How can this scenario be managed most effectively?

  1. Initiate rest, ice the iliac crest, and return to play when he is not limping.

  2. Initiate rest, ice the iliac crest, and return to play after 20 minutes.

  3. Keep the player on the sideline, perform a cognitive evaluation, and repeat the physical assessment.

  4. Keep the player out of the game and send him emergently to the hospital for imaging.

 

PREFERRED RESPONSE: 3

Question 25-Sideline examination of this patient showed no cervical pain or tenderness; motor and sensory function were normal; and his pupils were equal, round, and reactive. He was alert and oriented to the score of game, time on the clock, and current quarter of play. His iliac crest had mild tenderness but no swelling or crepitus. The player states that he has a slight headache and is no longer dizzy. What is the most appropriate treatment?

  1. Return him to the game and observe his play closely.

  2. Do not return to the game and do not allow play for the remainder of the season.

  3. Do not return to the game and begin a graduated return-to-play protocol for future games.

  4. Perform a sideline noncontact exercise testing examination and return him to the game if he is asymptomatic.

PREFERRED RESPONSE: 3

DISCUSSION FOR QUESTIONS 24 AND 25

Although this player limps off the field, the fact that he felt dizzy, had a headache, and did not initially recognize that he was playing in the third quarter indicates that he sustained a concussion. The player should be kept out of the game until a cognitive examination and repeat physical assessment is completed.Even if his physical symptoms have resolved, a certain period of time has expired, or he states that he is“ready,” he should not be returned to play prior to this assessment. Sending the patient to an emergency department should be considered only after this assessment and appropriate initial sideline treatment is initiated. The Consensus Statement on Concussion in Sport recommends that no athlete with concussion symptoms be returned to same-day play. This patient still has a slight headache, but even if this resolved he should not return to the game. Adolescents and high school athletes may have neurophysiological deficits that may not be evident on the sideline, or they may have a delayed onset of symptoms. A graduated return to play for future games is recommended.

 

CLINICAL SITUATION FOR QUESTIONS 26 THROUGH 29

A 32-year-old woman has a 2-year history of progressively worsening right groin pain that is exacerbated by activity. She reports no traumatic injury and an extensive work-up by her gynecologist has ruled out an intrapelvic source of her pain. The patient is a recreational athlete and exercises regularly in the gym.The pain is preventing her from performing these activities. She reports no catching or locking symptoms.Her examination reveals a physically fit female (BMI of 20) with limited right hip range of motion. She has no tenderness to palpation around the hip. While lying supine and bringing her hip into progressive flexion with internal rotation and adduction, her groin pain is reproduced. She has normal limb lengths and demonstrates weakness secondary to pain with hip flexion on the affected side.

Question 26-What is the most likely cause of this patient’s groin pain?

  1. Femoroacetabular impingement (FAI)

  2. Osteoarthritis of the sacroiliac joint

  3. Intra-articular loose body

  4. Trochanteric bursitis

PREFERRED RESPONSE: 1

Question 27-The patient is enrolled in physical therapy for 6 weeks with little improvement of her hip symptoms.What is the next most appropriate diagnostic test to determine the presence of an associated acetabular labral tear in this patient?

  1. Diagnostic arthroscopy of the hip

  2. MRI scan of the hip

  3. MRI arthrogram of the hip

  4. Ultrasound of the hip

PREFERRED RESPONSE: 3

 

Question 28-The study obtained in question 27 confirms the presence of an anterosuperior acetabular labral tear and pincer morphology of the acetabulum. What is the most likely location of a chondral injury associated with these findings?

  1. Posteroinferior acetabulum

  2. Posterosuperior acetabulum

  3. Femoral head above the fovea

  4. Femoral head below the fovea

PREFERRED RESPONSE: 1

Question 29-The patient experienced little improvement with activity modification and physical therapy. An intraarticular corticosteroid injection provides excellent but short-lived pain control. She requests surgical treatment for her hip and she is counseled regarding arthroscopy and consent is obtained. Intraoperatively,a capsulolabral separation is observed with an underlying pincer lesion. No articular cartilage injury is seen. What treatment is most appropriate considering these findings?

  1. Suture anchor repair of the labral tear and no bony resection

  2. Suture anchor repair of the labral tear and bony resection of the pincer lesion

  3. Debridement of the labral tear and bony resection of the pincer lesion

  4. Debridement of the labral tear with no bony resection of the pincer lesion

PREFERRED RESPONSE: 2

 

DISCUSSION FOR QUESTIONS 26 THROUGH 29

The clinical scenario, examination, and MRI scans are consistent with a pincer-type FAI. The decreased range of motion is secondary to the pain produced by the continued abutment of the femoral head against the anterosuperior acetabulum. Flexing the hip while internally rotating and adducting the leg recreates this contact and is typically painful. No clinical signs suggest sacroiliac joint arthritis, an intra-articular loose body, or trochanteric bursitis, although these are all diagnoses that should be considered in a patient with a painful hip. The most sensitive and specific study to detect an acetabular labral tear is an MRI arthrogram of the hip. This study should be obtained in this patient to evaluate the labrum as well as the status of the articular cartilage. An MRI scan without intra-articular contrast is not as sensitive as an arthrogram. An ultrasound can provide a dynamic assessment of the hip and help in the setting of a snapping hip; however, this study is not reliable to determine the presence of a labral tear. In the setting of pincer FAI, the forced leverage of the anterosuperior femoral head upon the anterior acetabulum results in abnormal forces against the posteroinferior acetabulum. This continued force can lead to a chondral lesion in this location know as a “counter-coup” injury. Chondral lesions of the femoral head are rare in the setting of pincer FAI. The posterosuperior quadrant does not experience increased force and rarely sustains chondral injuries. The patient is a young, active individual with no pre-existing degenerative changes, so repair of the tear with bony resection of the pincer lesion is the most appropriate treatment.A capsulolabral detachment should be repaired because these tears can heal and the labrum functions as a seal, preventing egress of synovial fluid from the joint space. If the pincer lesion is not resected, the patient will continue to experience abnormal contact and the repair will likely fail. There is no evidence that the patient has a cam impingement, and recontouring of the femoral head/neck junction is not appropriate. Simple debridement should be reserved for intrasubstance tears of the labrum, which would not be expected to heal with repair.

 

CLINICAL SITUATION FOR QUESTIONS 30 THROUGH 32

Figures 30a and 30b are the radiographs of a 20-year-old college multisport athlete who has had longstanding pain in his left hip. He denies any specific event that initiated his pain, but he notes that he had hip problems when he was an infant. He denies pain with activities of daily living, but he

 

believes his pain is increasingly limiting his ability to exercise. He localizes the pain to his groin. He denies low-back or buttock pain or pain that radiates down his leg.

 

 

Question 30-What examination findings are most consistent with the pathology seen in the radiographs?

  1. Pain with resisted hip flexion

  2. Pain with a half sit-up, plus tenderness at the pubic ramus

  3. Pain with a combination of hip flexion, adduction, and internal rotation

  4. Tenderness to palpation at the greater trochanter

PREFERRED RESPONSE: 3

 

Question 31-What is the most likely diagnosis for the source of this patient’s pain?

  1. Cam-type femoroacetabular impingement

  2. Pincer-type femoroacetabular impingement

  3. Hip flexor strain

  4. Athletic pubalgia

PREFERRED RESPONSE: 1

Question 32-Images from an MRI scan of this patient’s left hip are shown in Figure 30c through 30e. What is the most likely cause of his acute pain?

 

 

 

  1. Significant cartilage loss on the acetabulum

  2. Labral tear

  3. Femoral neck stress fracture

  4. Tendonopathy of the rectus femoris

PREFERRED RESPONSE: 2

 

DISCUSSION FOR QUESTIONS 30 THROUGH 32

This patient has cam-type femoroacetabular impingement. Decreased internal rotation and a positive impingement test (forced flexion, adduction, and internal rotation) are classic findings. The lack of pain with resisted hip flexion makes hip flexor strain unlikely, and the lack of tenderness at the greater trochanter renders trochanteric bursitis unlikely. Although athletic pubalgia can be a source of longstanding groin pain, he lacks the pain with a resisted sit-up and tenderness along the pubic ramus that is frequently noted in patients with pubalgia. His radiographs reveal a focal femoral neck prominence consistent with cam impingement, although pistol grip deformities and flattening of the lateral femoral head are often present as well. His MRI scan shows a labral tear, which is common in cam impingement.Surgical treatment for cam impingement can be effective for symptomatic patients. Even among highlevel athletes, open surgical dislocation of the hip has been shown to have good results. Most patients with cam impingement can be treated with arthroscopic osteoplasty and achieve results comparable to those realized with open surgical dislocation. The literature describes success in terms of athletes returning to sports (even professional athletes) to be approximately 90% after arthroscopic treatment.Byrd and Jones described 5 patients who developed transient neurapraxias that resolved uneventfully.The patients in his series who had concomitant microfracture had a 92% return to sports within the follow-up period. Cam impingement has long been thought to be associated with a history of a slipped capital femoral epiphysis. The capitis in these patients is displaced posteriorly, resulting in a prominent anterior femoral neck and decreased hip internal rotation. Pincer impingement is associated with a deep acetabulum, such as protrusion acetabula and acetabular retroversion. A patient who underwent a periacetabular osteotomy can develop a more retroverted acetabulum as well.

 

Question 33-A 25-year-old recreational soccer player has recurrent shoulder dislocations. He first dislocated his shoulder playing football in high school, was treated in a sling for 6 weeks, and returned to play for the remainder of the season. He did well until 2 years later when he reinjured the shoulder. He says that his shoulder dislocates with little injury and always “feels loose.” Examination reveals anterior instability and an MR arthrogram reveals an anterior-inferior labral tear and surgical treatment is recommended. He inquires about the benefits of arthroscopic vs open procedure. Which of the following statements reflects an advantage associated with arthroscopic procedures compared to open stabilization?

 

  1. Range of motion might be slightly better after an arthroscopic procedure.

  2. Rate of recurrent instability is lower after an arthroscopic procedure.

  3. Rates of return to work are higher after an arthroscopic procedure.

  4. Rates of return to sports are higher after an arthroscopic procedure.

 

DISCUSSION--There is much debate in the literature regarding optimal techniques for treatment of shoulder instability.Most studies have suggested a slightly better range of motion of the shoulder after an arthroscopic repair.Recurrent instability rates have been slightly higher with arthroscopic procedures in some studies, while others show the rates are not statistically different. Return to work and/or sports has been shown to be equal or slightly better with open procedures. PREFERRED RESPONSE: 1

 

Question 34-Figures 34a and 34b are the radiographs of a 38-year-old woman who had increasing left hip pain with activity. She noted no lower back or buttock pain and no pain along her lateral thigh. The pain usually only bothers her with running and cycling.

Nonsteroidal anti-inflammatory drugs helped initially but are not relieving her pain now. Examination with the patient supine reveals pain with internal and external rotation of her hip

 

with her hip and knee in an extended position. With her hip flexed to 90 degrees, she has internal rotation only to neutral, but full external rotation. What is the most likely diagnosis?

 

 

  1. Cam-type femoroacetabular impingement

  2. Pincer-type femoroacetabular impingement

  3. Intra-articular loose body

  4. Snapping psoas tendon

 

DISCUSSION -This patient has pincer femoroacetabular impingement. Her examination demonstrates pain with internal and external rotation of the femoral head in the acetabulum, suggesting intraarticular pathology.She also has a noticeable loss of internal rotation. Her examination findings make the other possible diagnoses unlikely. Her radiographs show a crossover sign, which suggests overcoverage by the anterior acetabulum, often the result of acetabular retroversion.

PREFERRED RESPONSE: 2

 

Question 35-A 24-year-old former high school wrestler had anterior cruciate ligament (ACL) reconstruction with hamstring autograft 6 years ago. He now experiences daily instability of his knee with routine activities including walking. Examination reveals a grade 3+ Lachman with a soft endpoint, varus laxity at 30 degrees, and a positive dial test at 30 degrees that dissipates at 90 degrees of knee flexion. He has mild medial joint line tenderness. When walking, there is a slight varus thrust. What treatment is most likely to lead to a successful outcome?

  1. Hamstring autograft

  2. Revision ACL reconstruction and posterior cruciate ligament (PCL) reconstruction

  3. Revision ACL reconstruction and posteromedial corner reconstruction

  4. Revision ACL reconstruction and posterolateral corner reconstruction

 

DISCUSSION -This patient underwent an ACL reconstruction that has now failed. Based on his examination, he also has a posterolateral corner injury. Because this concomitant injury was not treated, the patient had undue strain on his graft, resulting in ultimate failure. Hamstring grafts are as effective as other graft types for ACL reconstruction. The medial meniscus provides secondary stabilization to the knee; however, this patient has a missed lateral ligamentous injury, and meniscus tears do not result in the development of a varus thrust. An unrecognized PCL tear likely results in mild-to-moderate medial and patellofemoral osteoarthritis without significant lateral laxity and thrust. PREFERRED RESPONSE: 4

 

Question 36-A 49-year-old man is seeking a second opinion for continued knee pain and swelling. He went to his primary doctor for swelling “on top of his knee,” and he says his doctor drained some clear fluid. He noted that his condition improved for about 1 week before the swelling returned. He now has increasing pain and redness around his kneecap.

Examination reveals significant swelling of his prepatellar bursa,with erythema over the bursa that extends to the surrounding skin. His temperature in the office is 101.7°F. What is the next step in treatment for this patient?

 

  1. Initiate oral antibiotics for 7 days.

  2. Reaspirate the bursa and inject a corticosteroid.

  3. Recommend padding the patella for kneeling and ice.

  4. Perform an open bursectomy and start intravenous antibiotics.

 

DISCUSSION --This patient has septic prepatellar bursitis. Padding, cold therapy, compression, nonsteroidal anti-inflammatory drugs, and aspiration are thought to be acceptable treatments for aseptic prepatellar bursitis;they have little role in septic bursitis. Antibiotics along with aspiration or placement of a percutaneous drain have been associated with success in some cases of septic prepatellar bursitis, but the standard treatment is complete bursectomy with systemic antibiotics. PREFERRED RESPONSE: 4

 

CLINICAL SITUATION FOR QUESTIONS 37 AND 38

An 18-year-old right-hand-dominant college freshman who is a third baseman has pain in his right shoulder after using his outstretched right arm for support while diving to catch a low line drive. He describes pivoting on his right hand and arm while reaching out to make the catch with his left-hand glove. He had pain in his shoulder but was able to finish the game with some pain while throwing. Five days later, he experiences popping pain deep in his shoulder that has improved since the injury but continued to be bothersome deep in the shoulder with higher-velocity throwing. Examination demonstrated decreased internal rotation, posterior pain with cross-body adduction and posteriorly directed force, and full rotator cuff strength. Radiograph findings were normal. After 6 weeks of physical therapy (PT), his range of motion has improved but he continues to experience deep pain with therapy. Examination shows symmetric range of motion and posterior pain with the jerk test and Kim test. His rotator cuff is strong.Figure 37a Figure 37b

 

 

Question 37-What is the next step in treatment?

 

  1. Continued PT

  2. Subacromial injection

  3. CT scan

  4. MRI arthrogram

 

PREFERRED RESPONSE: 4

 

Question 38-T1-weighted, fat-saturated MRI scans are shown in Figures 37a and 37b. What is the next step intreatment?

 

  1. Open reduction internal fixation (ORIF)

  2. Rotator cuff repair

  3. Labrum repair

  4. Chondroplasty

PREFERRED RESPONSE: 3

 

DISCUSSION FOR QUESTIONS 37 AND 38

Examination findings of posterior glenohumeral tenderness, decreased internal rotation, and reproduction of symptoms with a posterior stress test indicate a posterior shoulder injury or instability. The jerk test,with the patient seated, positions the arm in forward flexion and internal rotation with elbow flexion. One hand of the examiner is placed on the patient’s distal clavicle and scapular spine and the other hand grasps the elbow. The arm is jerked posteriorly while the shoulder girdle is jerked anteriorly, which creates pain as the posteriorly subluxated humeral head relocates into the glenoid fossa. During the Kim test, the patient is seated with the arm in 90 degrees of abduction. While the arm is elevated 45 degrees diagonally (forward flexion and adduction), the examiner applies an axial load to the elbow and a downward and posterior force to the upper arm. A positive result causes a sudden onset of posterior shoulder pain. A positive jerk test combined with a positive Kim test has a 97% sensitivity for posterior instability. After extensive PT, the patient continues to have examination findings consistent with posterior shoulder injury or instability, so an MRI scan or MRI arthrography would be helpful to assess for any pathology.A subacromial injection is not indicated by this examination, which shows a strong rotator cuff and no demonstrated bursal-sided symptoms. A CT scan can be helpful in scenarios involving bony pathology,but an MRI is indicated at this stage in the evaluation of soft tissue. Although continuing PT may help to abate symptoms, the patient was continuing to have symptoms with PT. The MRI arthrogram shows a complex posterior labrum tear at the inferior to mid glenoid with separation of labrum from the glenoid. Because the examination findings are consistent with the MRI findings and nonsurgical treatment has failed to resolve symptoms, the next step is to recommend surgical treatment with labrum debridement and/or repair. No examination or MRI findings indicate a need for injection, rotator cuff repair, ORIF, or chondroplasty unless incidental intrasurgical findings are found.

 

Question 39A high school athlete sustained a noncontact injury to his right knee. He says that during a football game he felt a pop and his leg gave way. He attempted to continue to play but was unable secondary to pain.Five days after the injury, radiographs of his right knee do not reveal any abnormalities. On examination,he has an effusion on the injured side and no joint line tenderness. His range of motion is full extension to 110 degrees of flexion. At 20 degrees of flexion, he has increased anterior translation compared to the contralateral, uninjured left side. At 90 degrees of flexion, the tibia does not translate posteriorly. As his knee is moved from full extension into flexion with an internal rotation and valgus force, you notice a“clunk” within the knee. What is the most likely biomechanical basis for the “clunk”?

 

  1. In extension, the medial tibial plateau is subluxated; as an internal rotation/valgus force is applied in conjunction with flexion, the medial tibial plateau reduces.

  2. In extension, the medial tibial plateau is reduced; as an internal rotation/valgus force is applied in conjunction with flexion, the medial tibial plateau subluxates.

  3. In extension, the lateral tibial plateau is reduced; as an internal rotation/valgus force is applied in conjunction with flexion, the lateral plateau subluxates.

  4. In extension, the lateral tibial plateau is subluxated; as an internal rotation/valgus force is applied in conjunction with flexion, the lateral plateau reduces.

 

DISCUSSION -This patient sustained an isolated anterior cruciate ligament (ACL) injury based upon the mechanism described and examination findings. Increased anterior translation at 20 degrees of flexion (a positive Lachman test result) indicates an incompetent ACL. His other examination findings do not indicate a torn posterior cruciate ligament or torn menisci. The examination finding that produces the “clunk” has been termed the pivot shift maneuver and is positive in a knee with an incompetent ACL. Much has been written about the pivot shift examination and the pathologic motions that occur during this test. With an ACL-deficient knee in full extension and internal rotation, the lateral

 

tibial plateau subluxates anteriorly.When a valgus load is applied to the knee, the lateral plateau impinges on the lateral femoral condyle. As the knee is flexed, the lateral tibial plateau slides posteriorly into a reduced position, causing an audible clunk. Response 4 correctly describes the pathomechanics that result in the audible clunk heard during the pivot shift maneuver. Responses 1 and 2 are incorrect because they describe the medial tibial plateau,which is not part of the pathomechanics of the pivot shift. Response 3 is incorrect because in extension,the lateral tibial plateau is subluxated, not reduced. PREFERRED RESPONSE: 4

 

CLINICAL SITUATION FOR QUESTIONS 40 THROUGH 42

A 9-year-old boy was injured while playing soccer. His examination revealed painful range of motion between 5 degrees and 75 degrees. There is tenderness on the medial side of his knee. There is no effusion, a grade 1A Lachman, and severe pain over the medial epicondyle of the knee. Varus stress is negative and pain is elicited with valgus stress. Initial radiographs were negative for abnormality.

 

Question 40-What is the next diagnostic step?

  1. Repeat radiographs while the patient is weight bearing

  2. Ultrasound of the lower extremity and calf

  3. Stress radiographs

  4. CT scan

PREFERRED RESPONSE: 3

 

Question 41-Stress radiographs show a 2-mm medial physeal widening with valgus stress. What is the best initial treatment strategy for this patient?

  1. Femoral medial collateral ligament repair, extraphyseal

  2. Arthroscopically assisted medial collateral ligament repair

  3. Crutch ambulation without immobilization and weight bearing as tolerated

  4. Protected weight bearing with cast immobilization

 

Question 42-What is the most likely area of injury?

  1. Femoral attachment of the medial collateral ligament

  2. Tibial attachment of the medial collateral ligament

  3. Hypertrophic zone of the growth plate

  4. Proliferative zone of the growth plate

PREFERRED RESPONSE: 4

 

PREFERRED RESPONSE: 3

DISCUSSION FOR QUESTIONS 40 THROUGH 42

This patient likely has a physeal injury to the distal femoral physis. Stress radiographs or an MRI scan will most reliably reveal this diagnosis. The growth plate, when injured, is most commonly fractured through the hypertrophic zone of cartilage, its weakest point. This patient is optimally treated in a cylindrical or long-leg cast. Younger patients can be treated with a hip spica with a leg extension.

 

Question 43-An otherwise healthy 15-year-old wrestler has a 6-cm cutaneous lesion on the posterior aspect of his right elbow that he reports as a spider bite. What is the most likely diagnosis?

 

  1. Psoriasis

  2. Tinea corporis

  3. Herpes simplex virus

  4. Community-acquired methicillin-resistant Staphylococcus aureus (CA-MRSA)

 

DISCUSSION-Patients who have skin and soft-tissue infections caused by CA-MRSA often describe the lesion as a spider bite. The cytotoxin Panton-Valentine leukocidin that is produced by many strains of CA-MRSA causes tissue necrosis, resulting in rapid development of an abscess and the appearance of a spider bite.Patients with psoriasis have thick, red skin with flaky, silver-white patches. Tinea corporis is a general term for a cutaneous fungal infection. The lesion appears as a well-demarcated erythematous plaque with a raised border and central hypopigmentation, giving it a ring-like appearance. Primary infection with herpes simplex virus can produce constitutional symptoms with burning, tingling, or stinging at the site. Grouped vesicles with clear fluid 1 to 2 mm in size form on an erythematous base and then rupture,leaving moist ulcers or crusted plaques.

 

 

PREFERRED RESPONSE: 4

 

CLINICAL SITUATION FOR QUESTIONS 44 AND 45

Figure 44 is the MRI scan of a 14-year-old soccer player who injured his right knee during a game.He describes feeling a “pop” and he needed help walking off the field. His knee is visibly swollen. A Lachman test demonstrates asymmetry with no endpoint.

 

Question 44-Range of motion of the knee is between 0 degrees and 70 degrees. What is the most appropriate treatment option?

  1. Nonweight-bearing activity with crutches

  2. Microfracture of the chondral defect

  3. Immediate anterior cruciate ligament (ACL) reconstruction

  4. Delayed ACL reconstruction

PREFERRED RESPONSE: 4

Question 45-The patient has no postsurgical complications and begins physical therapy rehabilitation. The boy and his parents stress they “want to get the therapy over with as fast as possible” to expedite his return to sports,and the surgeon and rehabilitation team consider their request. Compared to nonaccelerated rehabilitation,patients who follow an early accelerated rehabilitation protocol experience

  1. increased laxity.

  2. no differences in long-term results.

  3. increased risk for graft failure.

  4. lower Knee Injury and Osteoarthritis Outcome Scores (KOOS).

PREFERRED RESPONSE: 2

 

DISCUSSION FOR QUESTIONS 44 AND 45

The Lachman test is the most sensitive examination for acute ACL injuries. ACL injury rates are higher in women than in men. This likely is attributable to anatomic differences (smaller notches, smaller ligaments, and different landing biomechanics). Lateral meniscal tears are more common than medial tears. The anterior drawer test is accentuated with 30 degrees of external rotation of the tibia. The MRI scan shows bone bruises consistent with an ACL tear. It is also always necessary to examine other structures of the knee in patients with ACL tears. All ligaments of the affected knee should be assessed.Lateral meniscal tears are frequently associated with ACL tears. Medial meniscal tears and posterolateral corner deficiency are also possible. Randomized clinical trials comparing an early accelerated vs nonaccelerated rehabilitation have demonstrated no significant differences in long-term results. These studies did not address timing of return to play with an early accelerated rehabilitation program. At 2 and 3 years postsurgically, there are no differences in laxity, number of graft failures, or KOOS scores.

 

Question 46-Chronic traumatic encephalopathy (CTE) is a neurodegenerative disease that is characterized by

  1. onset most often by age 30.

  2. a temporary state of neuronal and axonal derangement.

  3. manifestations of affect such as apathy, irritability, and suicidal ideation.

  4. absence of gross pathological brain changes upon autopsy.

 

DISCUSSION -CTE is a neurodegenerative disease that occurs years or decades after recovery from acute or postacute effects of head trauma. The exact relationship between concussion and CTE is not entirely clear; however,early behavioral manifestations of CTE have been described by family and providers to include apathy,irritability, and suicidal ideation. For some patients, cognitive difficulty such as poor episodic memory and executive function may be the first signs of CTE. Onset most often occurs in midlife after athletes have completed their sports careers, with mean age of onset at 42 years. The effects on the brain are degenerative, leading to a permanent state of derangement.

Autopsy findings demonstrate multiple gross pathological findings. The condition is more common among contact athletes. PREFERRED RESPONSE: 3

 

Question 47-In the shoulder position of 90-degree forward flexion and internal rotation, what is the most important static stabilizer of the glenohumeral joint?

  1. Rotator interval

  2. Infraspinatus

  3. Anterior band of the inferior glenohumeral ligament

  4. Posterior band of the inferior glenohumeral ligament

 

DISCUSSION -In the position of 90 degrees forward flexion and internal rotation, the most important static stabilizer of the glenohumeral joint is the posterior band of the inferior glenohumeral ligament. This position places the posterior-inferior glenohumeral ligament in an anterior-posterior direction and under tension. The superior glenohumeral ligament and the middle glenohumeral ligament provide static stability in the fully adducted and midrange-adducted positions, respectively. The subscapularis and infraspinatus provide primarily dynamic stability to the glenohumeral joint. Though not fully clear, the rotator interval appears to provide more static stability with the arm adducted, limiting inferior and posterior translation, and less so in the forward flexion and internal rotation position.

PREFERRED RESPONSE: 4

 

CLINICAL SITUATION FOR QUESTIONS 48 AND 49

A 17-year-old high school wrestler sustains an abrasion over the posterior aspect of his right elbow during a match. During the next few days the abrasion becomes erythematous and he is placed on oral cephalexin 500 mg four times per day. The erythema extends proximally despite the antibiotic regimen. The patient is afebrile, there is no fluid collection associated with the lesion, and his elbow joint is not involved.

 

Question 48-What is the most appropriate treatment?

 

  1. Switch to oral trimethoprim-sulfamethoxazole double-strength twice per day for 10 to 14 days

  2. Switch to oral ciprofloxacin 500 mg twice per day for 10 to 14 days

  3. Begin cefazolin 1 gram intravenously (IV) every 8 hours for 7 to 10 days

  4. Irrigation and debridement with empiric IV antibiotic coverage

 

PREFERRED RESPONSE: 1

 

Question 49-Assuming that the lesion can be covered appropriately and there is no drainage from the lesion, when should the patient be allowed to safely return to wrestling?

  1. When the absence of pain is reported by the wrestler for 3 consecutive days

  2. When 72 hours of antibiotics have been administered and there is no extension of the lesion for 48 hours

  3. When laboratory values are within defined limits and the patient remains afebrile for 3 days

  4. When the lesion has decreased in size by 50%

 

PREFERRED RESPONSE: 2

DISCUSSION FOR QUESTIONS 48 AND 49

This patient has cellulitis, which is typically caused by group A Streptococcus or Staphylococcus. The patient’s lack of improvement with first-line antibiotics is concerning for methicillin-resistant Staphylococcus aureus (MRSA) infection. MRSA cellulitis is becoming more prevalent in young athletes,and a high index of suspicion is required to provide appropriate intervention during this

aggressive disease process. The diagnosis is typically made clinically without the use of cultures. Oral trimethoprimsulfamethoxazole (a sulfonamide-class drug) double strength twice daily for 10 to 14 days or doxycycline (a tetracycline-class drug) 100 mg twice daily for 10 to 14 days are recommended for first-line treatment of suspected MRSA cellulitis. There is no indication to proceed with irrigation and debridement; however, if the patient develops a soft-tissue abscess or the underlying joint becomes involved, this would be an appropriate intervention. Switching the athlete to an IV cephalosporin (cefazolin) is not likely to be effective against the presumed resistant bacteria.

Ciprofloxacin (a fluoroquinolone-class drug) is effective against many bacteria, but not MRSA. The current recommendation for wrestlers with cellulitis is that return to competition be allowed after 72 hours of antibiotic treatment if there has been no extension of the cellulitis for 48 hours, the lesion can be covered, and there is no drainage from the lesion. The other responses are not current recommendations for return to competition.

 

Question 50-A 19-year-old linebacker underwent a coracoid transfer procedure for recurrent anterior glenohumeral instability. At his 1-week postsurgical check-up, his incision is doing well; however, he reports numbness over the lateral aspect of his forearm. What nerve may have been injured during his surgery?

  1. Axillary

  2. Median

  3. Musculocutaneous

  4. Radial

 

DISCUSSION-This patient has sustained an injury to the musculocutaneous nerve. The terminal branch of this nerve is the lateral antebrachial cutaneous nerve of the forearm. The axillary nerve provides sensation to the lateral arm. The median nerve provides sensation more distally. The radial nerve is not likely to be injured with a coracoid transfer procedure; if it is, the injury would result in numbness near the wrist. PREFERRED RESPONSE: 3

 

Question 51-What is the most important genetic element that distinguishes community-acquired methicillin-resistant Staphylococcus aureus (CA-MRSA) from hospital-acquired MRSA?

  1. Beta-lactamase

  2. Penicillin-binding protein 2a

  3. Panton-Valentine leukocidin (PVL)

  4. Staphylococcus cassette chromosome (SCCmec) type I

 

DISCUSSION PVL is a cytotoxin that defines CA-MRSA and is not typical of hospital-acquired MRSA. PVL has the ability to lyse white blood cells and cause tissue necrosis, allowing for rapid progression of abscess formation. Beta-lactamase is an enzyme that breaks the beta-lactam bond of penicillin and is present in most strains of Staphylococcus aureus today. Synthetic penicillins such as methicillin are resistant to the effects of beta-lactamase. MRSA and CA-MRSA carry the mecA gene, which encodes a penicillinbinding protein with a very low affinity for beta-lactam antibiotics, resulting in methicillin resistance.SCCmec mobile genetic units carry the mecA gene with additional genetic elements that together yield the multidrug-resistant strains found in healthcare environments. SCCmec type IV is specific to CA-MRSA and lacks these additional genetic elements, resulting in less multidrug resistance. PREFERRED RESPONSE: 3

 

CLINICAL SITUATION FOR QUESTIONS 52 THROUGH 54

A 13-year-old baseball player fell while rounding second base 2 days ago. He said it felt like his knee buckled when he turned toward third base. He could not finish the game, but was able to bear weight with a limp. He has had no previous knee injuries, but now complains of pain in his right knee. Initial examination demonstrated an effusion, tenderness at the proximal medial collateral region and medial patellofemoral retinaculum, decreased range of motion, and patella apprehension. He was otherwise ligamentously stable. No other noteworthy physical findings were found.

 

 

Question 52-What is the next treatment step?

  1. Radiographs

  2. Arthroscopy

  3. MRI scan

  4. Duplex ultrasound

PREFERRED RESPONSE: 1

Question 53-What do the radiographs shown in Figures 52a and 52b reveal?

  1. Medial femoral chondyle physeal widening

  2. Osseous or osteochondral loose fragment

  3. Osgood-Schlatter disease

  4. Patella nondisplaced fracture

PREFERRED RESPONSE: 2

Question 54-Figures 52c and 52d show the proton density fat-saturated MRI scans. Treatment at this stage includes arthroscopy and

  1. early functional rehabilitation.

  2. proximal realignment alone.

  3. attempted internal fixation.

  4. medial collateral ligament (MCL) repair.

PREFERRED RESPONSE: 3

 

DISCUSSION FOR QUESTIONS 52 THROUGH 54

This patient’s examination indicates a patellar or peripatellar knee injury. Initial evaluation with radiographs will assess for fracture, subluxation, or osteochondral injury. Examination findings did not demonstrate a need for emergent surgery, an MRI scan, or an ultrasound, so radiographs are the initial diagnostic imaging choice. Radiographs show an osseous or osteochondral loose fragment.

There is no evidence of obvious nondisplaced fracture or physeal changes. In suspected patella dislocation or subluxation with loose fragment seen on radiographs, an MRI scan is indicated. Lateral release alone is seldom indicated in a knee that was normal before injury. Acute proximal realignment has not been shown to alter long-term outcomes for first-time dislocators. The examination and MRI scan did not indicate a need for MCL repair. Closed reduction of the osteochondral fragment would not be indicated or appropriate for this injury. Treatment should consist of arthroscopy or arthrotomy and attempted internal fixation of this fragment. If fixation is not possible, the loose body can be removed.

 

Question 55-While obtaining informed consent for a lateral closing-wedge osteotomy, what complication should be discussed with the patient as exclusive to this procedure and not encountered in medial opening-wedge osteotomy?

  1. Compartment syndrome

  2. Plate breakage

  3. Neurologic injury

  4. Proximal tibiofibular joint disruption

 

DISCUSSION -With lateral closing-wedge osteotomy, proximal tibiofibular disruption can occur. This is not seen in medial opening-wedge osteotomy. A technique has been developed to prevent this complication; a fibular osteotomy is performed at the same time as the tibial osteotomy. The other complications listed are seen in both techniques, with nonunion and plate breakage more common in opening-wedge high tibial osteotomy (HTO) and neurologic injury more common in closing-wedge HTO (with issues related to the common peroneal nerve most prevalent). Compartment syndrome is a devastating complication that can occur with any osteotomy, and a high index of suspicion should be maintained during the postsurgical course for patients who develop this condition.

PREFERRED RESPONSE: 4

 

CLINICAL SITUATION FOR QUESTIONS 56 THROUGH 58

 

 

Figure 56 is the MRI scan of a 15-year-old girl who had left knee pain after sustaining a noncontact twisting injury while playing soccer. She reported severe pain initially that has since improved. On examination, she had a large knee effusion with lateral joint line tenderness. Range of motion is from 5 degrees of extension to 70 degrees of flexion. She wishes to return to sports at her preinjury level of activity.

Question 56-What examination test is most likely to reveal abnormal findings?

  1. Pivot shift test

  2. Quadriceps active test

  3. Patellar apprehension test

  4. External rotation recurvatum test PREFERRED RESPONSE: 1

Question 57-What is the most appropriate treatment?

  1. Delayed ligament reconstruction

  2. Physical therapy and functional bracing

  3. Immediate ligament repair

  4. Immediate ligament reconstruction PREFERRED RESPONSE: 1

 

Question 58-What is the most likely mechanism of injury?

  1. External rotation

  2. Posterior translation

  3. Hyperextension and varus

  4. Anterior translation and internal rotation

PREFERRED RESPONSE: 4

DISCUSSION FOR QUESTIONS 56 THROUGH 58

The MRI scan shows a bone bruise of the lateral femoral condyle and lateral tibial plateau. This injury pattern is commonly associated with anterior cruciate ligament (ACL) rupture and an abnormal pivot shift test result. Treatment of an ACL tear in a high-demand athlete should consist of ligament reconstruction.In this patient, surgery should be delayed until she regains full range of motion to minimize risk for arthrofibrosis after surgery. Recent analysis has shown that the noncontact mechanism is more consistent with anterior translation, affecting both the medial and lateral compartments. The bone bruise in the lateral femoral condyle occurs more anterior than that of the medial femoral condyle, suggesting that internal rotation has occurred. The external rotation recurvatum test assesses for posterolateral corner injury, and a positive quadriceps active test is consistent with posterior cruciate ligament rupture. An abnormal patellar apprehension test result is suggestive of patellar instability. Nonsurgical treatment is unlikely to result in sufficient stability if this patient returns to sports at her preinjury level of activity.Primary ACL repair is associated with high failure rates. Although the precise mechanism of injury varies,injuries can be broadly classified into contact and noncontact injuries. Noncontact injuries occur with the knee in slight flexion, valgus, and internal rotation, and contact injuries typically involve a lateralside impact producing a valgus force to the knee. The valgus component of noncontact injuries has been thought to cause mainly lateral compartment bone bruising. Posterior translation is the most common mechanism of posterior cruciate ligament rupture, and hyperextension and varus is associated with posterolateral corner injury.

 

Question 59-A 12-year-old boy who plays multiple sports has had insidious-onset heel pain while running for 4 months. On examination, he had ankle dorsiflexion of 5 degrees. The squeeze test result was positive and the Thompson test result was negative. He has no pain with forced ankle plantar flexion. What is the most likely diagnosis?

 

  1. Achilles rupture

  2. Gastrocnemius strain

  3. Calcaneal apophysitis

  4. Os trigonum syndrome

 

DISCUSSION-Calcaneal apophysitis (Sever disease) is a common cause of heel pain in adolescent athletes who participate in running or jumping sports. The condition occurs primarily before or during peak growth and is characterized by a tight Achilles tendon, a positive squeeze test, and tenderness over the calcaneal apophysis. Pain is localized to the heel and exacerbated by running. Os trigonum syndrome involves posterior ankle impingement and is commonly associated with ballet dancers.

Gastrocnemius strain typically causes pain more proximally at the myotendinous junction. The Thompson test is performed with the patient lying prone on the examination table. Absence of ankle plantar flexion when the examiner squeezes the calf constitutes a positive test and is indicative of Achilles rupture.

PREFERRED RESPONSE: 3

 

CLINICAL SITUATION FOR QUESTIONS 60 AND 61

A 15-year-old boy has had shoulder pain for 4 weeks during the middle of baseball season. The patient says his pain is “all over my shoulder.” Examination reveals tenderness to palpation over the anterolateral aspect of the shoulder. Internal and external rotation range of motion is restricted.

Radiographs show metaphyseal sclerosis.

 

Question 60-What is the most significant risk factor for the development of little leaguer’s shoulder in this scenario?

 

  1. Recent increase in the number of pitches

  2. Gender (male)

  3. Glenohumeral internal rotation deficit

  4. Genetic factors

PREFERRED RESPONSE: 1

Question 61-This boy’s parents are eager to get him back on the field as soon as possible. What is the most appropriate treatment option?

  1. Screw fixation of the epiphysis

  2. Arthroscopic debridement

  3. A shut-down period until the boy is asymptomatic, and gradual return to pitching via a throwing program

  4. An intra-articular cortisone injection

PREFERRED RESPONSE: 3

DISCUSSION FOR QUESTIONS 60 AND 61

Although a recent increase in the number of pitches may have contributed to this patient’s development of little leaguer’s shoulder, the most significant overall factor is age. Little leaguer’s shoulder is caused by rotational stress placed on the proximal humeral epiphysis during overhead throwing. The growth plate is weakest to torsion stress, and is most susceptible to injury during periods of rapid growth commonly seen during puberty. Most chronic shoulder injuries occur in throwing athletes between 13 and 16 years of age. Genetic factors and gender have not been studied in association with little leaguer’s shoulder. An initial 3-month period of rest and activity modification will typically result in resolution of symptoms.Nonsteroidal anti-inflammatory drugs may be used as needed. After the rest period, a gradual return to baseline pitching is implemented until the patient is back to baseline. This protocol has a long-term success rate exceeding 90%.

 

 

 

Question 62-is an arthroscopic view of the intercondylar notch of a right knee from an anterolateral portal.What is the main function of the structure delineated by the black asterisks?

  1. Resist anterior translation during knee flexion

  2. Resist posterior translation during knee flexion

  3. Resist rotatory loads during knee flexion

  4. Resist rotatory loads during knee extension

 

DISCUSSION--The structure shown is the posterolateral bundle of the anterior cruciate ligament (ACL). This bundle is optimally positioned in the knee to resist rotatory forces during terminal knee extension. “Resist anterior translation during knee flexion” best describes the anteromedial

bundle. “Resist rotatory loads during knee flexion” is unlikely because the posterolateral bundle is tightest during knee extension. The posterior cruciate ligament, not the ACL, functions to resist posterior translation. PREFERRED RESPONSE: 4

 

Question 63-A 25-year-old wrestler has been experiencing increasing left knee pain since his last professional cage fight. He complains of both pain and instability on the medial side of his left knee. Examination reveals a grade 3 Lachman and pseudolaxity with valgus stress. Dial test findings are normal. Radiographs show medial degenerative changes and 5 degrees of varus alignment. What is the most appropriate treatment?

  1. Rehabilitation with vibration-platform weight-bearing squats

  2. Anterior cruciate ligament (ACL) reconstruction with autograft bone-tendon-bone

  3. High-tibial osteotomy (HTO)

  4. HTO plus ACL reconstruction at the same time

 

DISCUSSION-A young athlete with posttraumatic arthrosis of the knee isolated to the medial side poses a challenge.History and examination confirm key findings. Complaints of both pain and instability warrant concomitant HTO plus an ACL procedure, particularly in young athletes.

Rehabilitation with vibration platform is an unproven technique and is not specific to this knee problem. ACL reconstruction or HTO in isolation would not be ideal for this young, active patient. PREFERRED RESPONSE: 4

 

Question 64-What is an example of cognitive rest after concussion?

  1. Playing chess

  2. Increasing reading

  3. Working online

  4. Limiting video games

 

DISCUSSION--Physical and cognitive rest are recommended as treatment for sports-related concussion. Cognitive rest involves minimizing activities that require concentration and attention such as reading, schoolwork, video games, text messaging, working online, and playing games that require concentration such as chess.Limiting or eliminating video games after concussion is a form of cognitive rest. PREFERRED RESPONSE: 4

 

CLINICAL SITUATION FOR QUESTIONS 65 THROUGH 67

 

 

Figure 65 is the radiograph of a 24-year-old man who had left knee pain after sustaining a twisting injury while playing Ultimate Frisbee. On examination, he had a large effusion with tenderness over the lateral joint line. Pivot shift testing results were positive. Prone dial testing results at 30 degrees and 90 degrees were negative. An MRI scan shows a tear of the anterior cruciate ligament (ACL).

 

Question 65-Based on the radiograph, the attachment for which structure has been disrupted?

  1. Iliotibial band

  2. Popliteus tendon

  3. Lateral meniscus

  4. Lateral capsular ligament

PREFERRED RESPONSE: 4

Question 66-Compared with a transtibial technique, what effect will drilling the femoral tunnel through the anteromedial portal have?

  1. Produces a longer femoral tunnel

  2. Improves visualization while drilling

  3. Should be performed at 90 degrees of knee flexion

  4. Allows for independent access to the anatomic femoral insertion PREFERRED RESPONSE: 4

 

Question 67-The patient requests anatomic double-bundle ACL reconstruction. Compared with transtibial singlebundle ACL reconstruction, anatomic double-bundle ACL reconstruction is more likely to

  1. restore improved knee kinematics.

  2. allow for earlier return to sports.

  3. result in better clinical outcomes scores.

  4. be associated with lower surgery cost.

PREFERRED RESPONSE: 1

DISCUSSION FOR QUESTIONS 65 THROUGH 67

The radiograph shows a Segond fracture, an avulsion fracture involving the lateral capsular ligament.

This radiographic finding has been associated with ACL rupture in 75% to 100% of cases. Drilling the femoral tunnel through the anteromedial portal allows for independent access to the native femoral attachment. Fiber orientation is more oblique than with a transtibial technique and more closely resembles that of the native ligament. Double-bundle reconstruction attempts to duplicate native ACL anatomy.Biomechanical studies have shown that double-bundle reconstruction more

closely reproduces normal knee kinematics; however, this technique does not offer a clear advantage in terms of clinical outcomes.The iliotibial band inserts onto Gerdy’s tubercle. The popliteus tendon originates from the lateral femoral condyle. The lateral meniscus attaches near the intercondylar eminence at the anterior and posterior meniscal roots. Recent advances in ACL reconstruction focus on restoring the native ACL anatomy.Studies have determined that a knee flexion angle of 110 degrees is optimal to avoid blowout of the back wall and injury to the lateral structures while drilling.

Femoral tunnel length is typically shorter than with a transtibial approach and decreases with higher-flexion angles. Double-bundle reconstruction is associated with higher surgical costs because of the need for additional fixation and, in the case of allograft reconstruction, a second graft.

 

Question 68-A 42-year-old man has increasing pain and, to a lesser extent, some occasional left knee instability.Several years earlier he sustained a noncontact twisting injury to his knee. He had some initial soreness and pain but was able to resume his normal activities while avoiding sports. On examination, the patient has medial joint line pain, a grade 2+ Lachman, and a slight varus thrust. His radiographs reveal mild-tomoderate medial compartment osteoarthritis with varus alignment. What surgical treatment strategy likely will alleviate his pain?

  1. Distal femoral osteotomy

  2. Unicompartmental knee replacement

  3. High tibial osteotomy (HTO), lateral closing wedge

  4. HTO, medial opening wedge with decreased tibial slope

 

DISCUSSION-This patient had a previous anterior cruciate ligament (ACL) and posterolateral complex injury. With chronic instability and osteoarthritis, the best option is HTO with a decrease in the tibial slope to reduce anterior laxity. Distal femoral osteotomy is better suited to address valgus malalignment. The lateral closing-wedge osteotomy would not allow for adequate correction of the tibial slope. Unicompartmental knee replacement is not indicated when there is ligament instability. If the patient continues to experience instability following correction of the varus malalignment, reconstruction of the ACL and posterolateral corner would be appropriate at that time.

PREFERRED RESPONSE: 4

 

RESPONSES FOR QUESTIONS 69 THROUGH 71

  1. Bone-patella tendon-bone autograft (10 mm)

  2. Soft-tissue allograft (8 mm)

  3. Quadruple hamstring autograft (10 mm)

 

For each of the following, please select the preferred response from the choices above.

Question 69-Shortest biologic incorporation after surgical reconstruction

PREFERRED RESPONSE: 1

Question 70-Highest ultimate tensile load (UTL) Question 71-Highest risk for disease transmission

 

PREFERRED RESPONSE: 3

 

PREFERRED RESPONSE: 2

DISCUSSION FOR QUESTIONS 69 THROUGH 71

Anterior cruciate ligament (ACL) graft incorporation into bone follows a multiphase pattern. The first phase is an inflammatory response with initial donor cell degeneration and provision of a scaffold for host cell migration (occurs in up to 20 days). The second phase involves revascularization and host cell fibroblast migration (20 days to 6 months). The final phase involves completion of graft healing and remodeling into a more organized pattern of collagen structure. Bone-to-bone healing has been found to have the shortest duration at approximately 6 weeks. Multiple studies have evaluated the UTL of the intact ACL and various tissues used for ACL reconstruction. The quadruple hamstring autograft of equivalent diameter has the highest UTL of the examples given at approximately 4000 N. In comparison,the native ACL and the bone-tendon-bone autograft have a UTL of approximately 2100 N and 3000 N, respectively. Allograft carries the highest risk for disease transmission among the examples in this question. These risks are low and largely eliminated with the screening guidelines developed and updated by the American Association of Tissue Banks.

 

 

 

Question 72-Figure 72 is the MRI scan of a 61-year-old man who had left shoulder pain with a massive rotator cuff tear. Active forward elevation was 120 degrees. Arthroscopic examination revealed that the rotator cuff tear was irreparable. The articular surfaces of the glenohumeral joint have a normal appearance without significant degenerative changes. What is the most appropriate treatment option?

 

  1. Biceps tenotomy

  2. Loose body removal

  3. Latissimus dorsi transfer

  4. Reverse total shoulder arthroplasty

 

DISCUSSION -The MRI scan shows medial subluxation of the biceps tendon. Biceps tenotomy has been an effective treatment option for patients with large to massive rotator cuff tears when the tear is irreparable and pain is the main symptom. The MRI scan does not show a loose body. Patients with severe external rotation

deficit and a deficient teres minor may experience a better functional result with latissimus dorsi transfer.Reverse total shoulder arthroplasty is an option in patients with cuff tear arthropathy and pseudoparalysis. PREFERRED RESPONSE: 1

 

CLINICAL SITUATION FOR QUESTIONS 73 THROUGH 75

A 17-year-old female swimmer has a 3-month history of increasing right shoulder pain. She reports feeling as if her shoulder is "popping" out of place and pain with overhead activities. She also reports nonspecific sensory changes in her arm on rare occasions. She has had to stop all sports activities because her symptoms have progressed. On examination, sulcus and apprehension sign findings are positive. She also has a positive Neer impingement sign. The Jobe empty can test and speed test results are negative.

 

Question 73-What structure is most likely associated with the etiology of this patient’s symptoms?

  1. Rotator cuff

  2. Biceps tendon

  3. Capsule

  4. Brachial plexus PREFERRED RESPONSE: 3

Question 74-What is the most appropriate initial treatment for her condition?

  1. Complete rest and a slow return to sports

  2. Physical therapy

  3. Corticosteroid injection

  4. Arthroscopic surgery PREFERRED RESPONSE: 2 Question 75-After 1 year of following recommended treatment, this patient continues to experience her symptoms and has had to cease all sports activity. An MRI scan reveals no evidence of definitive labral or rotator cuff pathology. At this stage, what is the most appropriate treatment option?

  1. Arthroscopic rotator cuff repair

  2. Arthroscopic biceps tenodesis

  3. Arthroscopic capsular plication

  4. Arthroscopic superior labral anterior-posterior repair

PREFERRED RESPONSE: 3

DISCUSSION FOR QUESTIONS 73 THROUGH 75

This patient has a history most consistent with multidirectional instability. A lax capsule causes subluxation of the shoulder and strain on the rotator cuff and may result in pain and instability. The capsule is most closely associated with the cause of her problem. Initial treatment for multidirectional instability is physical therapy focusing on restoring balance to the shoulder with rotator cuff and scapular stabilization exercises. Nonsurgical therapy should be protracted and is the mainstay of treatment in this scenario. This patient has exhausted all nonsurgical measures and is now a candidate for surgical reconstruction. Capsular plication will best address the lax capsule and provide the best option for reducing her symptoms. The rotator cuff and biceps tendon may be secondarily strained but are not the primary sources of the problem. The brachial plexus does not address the etiology, but rather the symptoms that may occur as a result of instability of the shoulder joint.

Complete rest will not alleviate the patient's underlying condition because the shoulder girdle may still be weak and symptoms likely will return. A corticosteroid injection and arthroscopic surgery are too invasive as initial treatment for this condition. Arthroscopic rotator cuff repair, a biceps tenodesis, and superior labral anterior-posterior repair are unlikely to result in symptomatic improvement for this patient and are not associated with pathologic findings in the setting of multidirectional instability.

 

Question 76-What do the T2-weighted, fat-saturated MRI scans shown in Figures 76a through 76d reveal?

 

 

 

  1. Posterior cruciate ligament (PCL) tear, isolated

  2. PCL tear and medial meniscus tear

  3. Anterior cruciate ligament (ACL) tear, isolated

  4. ACL tear and medial meniscus tear

 

PREFERRED RESPONSE: 4

DISCUSSION--The MRI scans show that edema is noted on the femoral insertion of the ACL consistent with a high-grade or complete ACL tear. The ACL is not visualized on the sagittal view, although the torn meniscus can be seen in the notch. On the coronal image, there is an empty lateral wall sign indicating proximal disruption of the ACL. The medial meniscus images show a disruption of normal meniscus morphology consistent with a bucket handle medial meniscus tear. Note the appearance on the sagittal MRI scan of what appears to be a second soft-tissue density in line with the PCL. This “double PCL” sign is highly indicative of a displaced medial meniscus tear rather than a displaced lateral meniscus tear.

 

Question 77-A 28-year-old woman underwent a closing-wedge high tibial osteotomy (HTO) for medial compartment overload after medial meniscectomy. Postsurgically, she reported improvement in her medial pain and resumed normal activities. About 9 months after her surgery, however, she reports burning pain in the front of her knee with running. Her examination reveals no joint line tenderness, mild pain with patellar compression, and limited patellar glides. What is the most likely cause of her symptoms?

  1. Patella infera (baja)

  2. Patella alta

  3. Recurrence of medial joint overload

  4. Nonunion of the osteotomy

 

DISCUSSION--After HTO, particularly in patients who have been immobilized after a closing-wedge osteotomy, patella baja is a common finding. This can precipitate anterior knee pain or patellofemoral pain syndrome.Recurrence of medial joint overload is incorrect because the patient has no medial joint complaints.Nonunion is less likely with a closing-wedge osteotomy and likely will not result in anterior knee pain. PREFERRED RESPONSE: 1

 

CLINICAL SITUATION FOR QUESTIONS 78 THROUGH 80

A 14-year-old pitcher who plays in an extended-season baseball program has vague pain in his dominant shoulder. The pain occurs with throwing and it has been worsening during the past 2 months. External rotation in abduction is 140 degrees in the dominant arm and 130 degrees in the nondominant arm.Internal rotation in abduction is 30 degrees in the dominant arm and 40 degrees in the nondominant arm.The sulcus sign is negative. Radiographs show subtle widening of the proximal humeral physis.

Question 78-What is the most likely diagnosis?

  1. Little leaguer’s shoulder

  2. Glenohumeral internal rotation deficit

  3. Rotator cuff tendonitis

  4. Multidirectional instability

PREFERRED RESPONSE: 1

Question 79-Initial treatment should consist of

  1. arthroscopic capsular plication.

  2. rest until symptoms resolve.

  3. manipulation under anesthesia.

  4. closed reduction and percutaneous pinning. PREFERRED RESPONSE: 2

 

Question 80-What is the etiology of this patient’s problem?

  1. Ischemia

  2. Overuse condition

  3. Pathologic laxity

  4. Increased humeral retroversion

PREFERRED RESPONSE: 2

DISCUSSION FOR QUESTIONS 78 THROUGH 80

Little leaguer’s shoulder is an overuse condition of the proximal humeral physis. Patients report diffuse pain that is worse with throwing. Factors that contribute to the condition include excessive throwing,improper throwing mechanics, and muscle-tendon imbalance. Radiographs usually show widening of the proximal humeral physis, and, in more severe cases, metaphyseal demineralization or fragmentation. Surgical fixation is not required for healing. Treatment involves rest until symptoms resolve, followed by initiation of an interval throwing program. Pitching coaches should evaluate throwing mechanics and maintain pitch counts. The dominant shoulders of throwing athletes undergo adaptive changes, resulting in increased external rotation and decreased internal rotation. These changes occur secondary to softtissue and bony adaptations, including increased humeral retroversion. Ischemia has been implicated as a potential cause of osteochondritis dissecans lesions. Rotator cuff tendonitis presents with anterolateral shoulder pain that is worse with activity. Pain is reproduced with resisted supraspinatus testing, and radiograph findings are typically normal.

Adolescent shoulder injuries are often caused by subtle, atraumatic instability most often sustained in sports with overhead movements, including baseball. These symptoms are more likely to occur in athletes with evidence of increased soft-tissue laxity.

 

Question 81-The sports-related concussion treatment guidelines regarding same-day return to play developed during the 2008 3rd International Conference on Concussion in Sport in Zurich, Switzerland, recommend that

  1. athletes with resolution of symptoms after an injury may return to play.

  2. athletes with resolution of symptoms after an injury may not return to play.

  3. a CT scan should be used for early diagnosis of concussion.

  4. a noncontact athlete with symptoms at rest or with exertion should continue to play.

 

DISCUSSION--The Consensus Statement on Concussion in Sport was developed at the 3rd International Conference on Concussion in Sport in Zurich in November 2008. Although the authors acknowledged that the science of concussion was evolving, consensus existed regarding the principal messages conveyed at this conference.Specifically, the panel agreed that athletes with symptoms should not be returned to play on the same day and that subsequent serial monitoring and re-examination should be performed. Even if symptoms resolve during a game, an injured athlete should not return to play on the same day. These guidelines apply to both minimal-contact and noncontact athletes. Panel members believed that evaluation and management recommendations in these guidelines could apply to children age 10 years and older. For children younger than age 10, reported concussion symptoms differ from those experienced by adults, and age-appropriate symptom checklists are required. Young elite athletes also should be treated with a more conservative approach. The use of CT scan in early diagnosis of concussion was considered to contribute little to concussion evaluation. However, if, based on serial examination, suspicion of an intracerebral structural lesion is suspected, a CT scan should be employed. A multidisciplinary approach including a sideline physician, trainers, parents, and consulting physicians was considered helpful and was recommended. PREFERRED RESPONSE: 2

 

 

 

Question 82-Figure 82 is the MRI scan of a 15-year-old boy who has had knee pain with running for 5 months. Radiographs show an osteochondritis dissecans (OCD) lesion of the medial femoral condyle. What is the most appropriate treatment?

 

  1. Arthroscopic or open reduction and internal fixation

  2. Arthroscopic loose body removal

  3. Activity restriction for up to 9 months

  4. Subchondral drilling

 

DISCUSSION--OCD is an acquired lesion of the subchondral bone. Patients with OCD initially report nonspecific pain and variable amounts of swelling. Initial radiographs help identify the lesion and establish the status of the physes. An MRI scan is useful for assessing the potential for the lesion to heal with nonsurgical treatment. Nonsurgical treatment is appropriate for small, stable lesions in patients with open physes and focuses on activity restriction for 3 to 9 months. Surgical treatment is necessary for

unstable or detached lesions. Stable lesions with intact articular cartilage can be treated with subchondral drilling to stimulate vascular ingrowth, with radiographic healing at an average of 4.4 months. Fixation is indicated for unstable or hinged lesions, and stabilization of the fragment can be achieved using a variety of implants through an arthroscopic or open approach. The fragment should be salvaged and the normal articular surface restored whenever possible. PREFERRED RESPO: 1

 

CLINICAL SITUATION FOR QUESTIONS 83 AND 84

A 6-ft, 6-in white man has undergone 3 previous open right-knee surgeries. He recently learned he had an intra-articular loose body and underwent an arthrotomy with loose body removal. At surgery, 2 previous skin incisions were noted: midline and medial parapatellar. The orthopaedic surgeon uses the midline incision for the arthrotomy and loose body removal. At the same time, the surgeon performs a lateral release. The patient required 110 minutes of tourniquet time at a pressure of 325

mm. Postsurgically, a cold compression device was applied over an island dressing and the patient continuously wore a thromboembolic-deterrent stocking for 3 days. Three days after surgery the patient complained of burning over the anterior aspect of his knee. He is afebrile and denies chills. Upon inspection, the wound has blanching over the anterior knee and several clear blisters are present. There is no erythema or purulent drainage from the wound. The knee has no effusion and good peripheral pulses are palpated.

 

Question 83-The appearance of the patient’s wound is most likely the result of

  1. a postoperative infection.

  2. a hypersensitivity reaction to the dressing.

  3. frostbite injury secondary to continuous cold exposure.

  4. frostbite injury with superimposed secondary infection.

PREFERRED RESPONSE: 3

Question 84-What is the most appropriate initial treatment?

  1. Run a wound culture and return to the operating room for arthrotomy, irrigation, and debridement.

  2. Return to the operating room, blister debridement, split-thickness skin grafting, and prophylactic intravenous (IV) penicillin G (PCN G) every 6 hours

  3. Return to the operating room, blister debridement, apply topical aloe, and prophylactic IV PCN G every 6 hours

  4. Bedside blister debridement, apply topical aloe, and prophylactic IV PCN G every 6 hours

 

PREFERRED RESPONSE: 4

DISCUSSION FOR QUESTIONS 83 AND 84

Based upon the history and description of the wound, this patient has sustained a frostbite injury to the anterior skin without evidence of infection. Frostbite, a thermal injury to local tissues, can be classified into first-, second-, third-, and fourth-degree injuries. First-degree injuries are characterized by a central whitish area surrounded by erythema. Second- and third-degree injuries are characterized by blisters that appear within the first 24 hours. Second-degree blisters are clear or cloudy, while third-degree blisters are hemorrhagic. Fourth-degree injuries are characterized by tissue necrosis. Treatment of a frostbite injury should begin as soon as it is identified, even if severity has not been determined. Treatment includes protection against mechanical irritation and keeping the injured area away from heat. The limb should be elevated and clear blisters debrided; dark blisters should be drained but not debrided. Topical treatment includes aloe application every 6 hours (silver sulfadiazine for open wounds). Nonsteroidal anti-inflammatory drugs can be given along with PCN G (500,000 U) or clindamycin (600 mg) IV every 6hours for 48 to 72 hours as prophylaxis against infection.Multiple skin incisions on the front of the knee place the skin at healing risk after surgery.

Skin oxygenation is further compromised by a lateral release, which disrupts the superior lateral geniculate artery in the peripatellar vascular anastomosis. When addressing compromised perfusion to the skin, the orthopaedic surgeon may consider not performing a lateral release, using the medial parapatellar incision,and/or not using a tourniquet during surgery. In this patient, the use of cold therapy further compromised perfusion to the skin. This patient’s wound does not reflect infection, but rather frostbite from the compromised skin perfusion and the application of cold therapy.

 

 

The physiologic responses to the effects of freezing temperatures on limbs have been categorized into 4 phases. Phase I, cooling and freezing, is characterized by vasoconstriction and vasospasm. The freezing results in mechanical destruction of cell membranes in endothelial cells in small capillaries. Phase II,rewarming, initiates as heat absorbed by the limb begins an exothermic reaction as extracellular and intracellular crystals melt. Intracellular swelling occurs and small capillaries become highly permeable,resulting in extravasation of fluid and causing edema and blisters. Phase III is characterized by progressive tissue injury resulting from inflammation, vascular stasis, and thrombosis leading to ischemia.Phase IV is recovery that can progress along 3 potential pathways: complete healing, healing with later sequelae, or early tissue necrosis leading to gangrene.

 

CLINICAL SITUATION FOR QUESTIONS 85 THROUGH 87

Figure 85 is the radiograph of a 13-year-old right-handed baseball player who has experienced 2 months of right-dominant shoulder pain. As a pitcher, he says he cannot “throw hard” without pain, and he develops a dull ache if he throws more than 15 pitches.

 

Question 85 -Where is this injury located?

  1. Hypertrophic zone of the physis

  2. Posterior superior humeral head

  3. Surgical neck of the humerus

  4. Resting cartilage zone of the physis

 

Question 86-What is the diagnosis?

  1. Osteonecrosis

  2. Proximal humerus epiphysiolysis

    PREFERRED RESPONSE: 1

  3. Hill-Sachs impaction fracture from subluxation

  4. Salter-Harris type II fracture of the proximal humerus

 

PREFERRED RESPONSE: 2

 

Question 87-What is the recommended treatment for this injury?

  1. Bankart repair

  2. Hanging arm cast

  3. Rest and no pitching for 3 months

  4. Resume a throwing program but avoid curveballs and sliders

PREFERRED RESPONSE: 3

DISCUSSION FOR QUESTIONS 85 THROUGH 87

The hypertrophic zone of the growth plate has been implicated as the weak link in the physis in acute injuries. Epiphysiolysis of the proximal humerus in throwing athletes occurs as the result of tension and shear on the physis. More than 90% of affected patients who are treated with rest for an average of 3 months become asymptomatic. Prevention is the best option. Set limitations of the number of pitches and types of pitches depending on the age of the player. Also recommend use of proper pitching mechanics.

 

Question 88-A 45-year-old postmenopausal smoker with a body mass index (BMI) of 22 has had severe knee pain for the past year. The pain has been progressing and the patient is now only able to perform activities of daily living. Knee radiographs reveal medial compartment osteoarthritis without any involvement of the patellofemoral joint or the lateral compartment. What is the contraindication for a high tibial osteotomy (HTO) in this patient?

  1. Smoking status

  2. Postmenopausal status

  3. BMI

  4. Radiographic findings

 

DISCUSSION- The principal contraindications to valgus-producing HTO include (1) lateral compartment degenerative joint disease, (2) loss of a significant portion of the lateral meniscus, (3) symptomatic patellofemoral degenerative joint disease, (4) nonconcordant pain (ie, patellofemoral pain with medial compartment osteoarthritis), (5) smoking, (6) patient unwillingness to accept the anticipated cosmetic appearance of the desired amount of angular correction, and (7) inflammatory arthritis. PREFERRED RESPONSE: 1

 

CLINICAL SITUATION FOR QUESTIONS 89 THROUGH 95

An 18-year-old high school basketball player sustains a noncontact injury to his right knee while landing after a rebound. He reports hearing a “pop” and has immediate pain and the inability to bear weight on that leg. During the next few hours he develops an effusion, which is aspirated by the team physician and is found to be a lipohemarthrosis. After aspiration, the athlete is not able to perform a straight-leg raise and a detailed examination is not possible because of guarding and pain.

 

 

 

Question 89-What is the next treatment step?

  1. Place the patient in a telescoping hinged knee brace that is unlocked and allow weight bearing as tolerated with crutches for support.

  2. Place the patient in a telescoping hinged knee brace locked in extension with toe-touch weight bearing.

  3. Place the patient in a neoprene sleeve and allow weight bearing as tolerated.

  4. Place the patient in a neoprene sleeve with toe-touch weight bearing.

PREFERRED RESPONSE: 2

Question 90-Radiographs are shown in Figures 89a through 89c. What is the most likely diagnosis?

  1. Medial tibial plateau fracture

  2. Chronic medial collateral ligament injury

  3. Anterior cruciate ligament (ACL) rupture

  4. Acute lateral collateral ligament rupture

PREFERRED RESPONSE: 3

 

 

Question 91-Figures 89d and 89e are this patient’s T2-weighted sagittal MRI scans. What is most commonly associated with these MRI findings?

  1. Medial meniscus tear

  2. Lateral meniscus tear

  3. Posterolateral corner (PLC) injury

  4. Posterior cruciate ligament (PCL) rupture

PREFERRED RESPONSE: 2

Question 92-This athlete’s MRI scan reveals an acute ACL tear. He has accepted a scholarship to play basketball at an NCAA Division I school. What is the role of the team physician in reporting this injury to the scholarship school?

  1. Encourage the athlete to report his injury and treatment to the scholarship school’s coaching staff.

  2. Encourage the athlete not to discuss the injury with his scholarship school because this might endanger his scholarship.

  3. Directly contact the scholarship school’s medical staff and report the injury and treatment rendered.

  4. Discuss the injury with the athlete’s parents and allow them to make a decision about how to proceed.

PREFERRED RESPONSE: 1

 

Question 93-The athlete is taken to the operating room for arthroscopic evaluation and treatment. While the patient is under anesthesia, the knee is found to have full motion with a grade 2B Lachman examination, a positive pivot shift, 1+ posterior drawer, and equivalent external rotation of the tibia in 30 degrees and 90 degrees of flexion. The examination is consistent with what injury?

  1. Isolated incomplete ACL rupture

  2. Complete ACL rupture

  3. Complete ACL rupture with posterolateral corner injury

  4. Isolated posterolateral corner injury

PREFERRED RESPONSE: 2

Question 94- When compared to double-bundle ACL reconstruction, what is the disadvantage of traditional trans-tibial single-bundle ACL reconstruction?

  1. Less anterior-posterior stability

  2. Less rotational stability

  3. Higher cost

  4. Longer surgical time

PREFERRED RESPONSE: 2

 

Question 95-Postsurgically, the patient recovers well and is fully rehabilitated. He demonstrates full motion with no instability or pain and is cleared to return to play 12 months after the surgery. He asks for your advice regarding use of a functional brace for playing basketball following his reconstruction. What is the most appropriate recommendation?

 

  1. The athlete must wear a functional brace for all athletic activities for 2 years following reconstruction.

  2. The athlete may wear a functional brace for athletic activities; however, no evidence exists to show the brace decreases the rate of ACL retear.

  3. The athlete must wear a functional brace for 2 years following reconstruction for basketball only; other athletic activities such as running and tennis are allowed without the brace.

  4. The athlete must wear a custom-fit functional brace for 2 years following reconstruction because off-the-shelf braces produce inferior results.

PREFERRED RESPONSE: 2

DISCUSSION FOR QUESTIONS 89 THROUGH 95

The athlete most likely suffered an acute ACL rupture however the presence of a lipohemarthrosis is concerning for the possibility of an intraarticular fracture. Because of this, the patient should not be allowed full weight bearing until a fracture is ruled out with radiographs. Given the athlete’s inability to perform a straight leg raise, the extensor mechanism is not functioning and a telescoping knee brace locked in extension should be utilized. A neoprene knee sleeve does not have a role in the

treatment of this acute injury.The radiographs reveal a lateral avulsion fracture off of the proximal tibial epiphysis which is known as a Segond fracture. It is indicative of an ACL injury and the fracture fragment seen is the consequence of the lateral capsule injury sustained during the pivot-shift mechanism. There is no radiographic evidence of a medial tibial plateau fracture. A radiographic sign of a chronic MCL injury is known as a Pellegrini-Stieda lesion and this is seen as calcification of the femoral origin of the MCL. A radiographic sign of an acute LCL rupture would be an avulsion fracture of the tip of the fibula.The MRI shows kissing contusions of the posterolateral tibial plateau and the midpoint of the lateral femoral condyle. These “kissing lesions” are seen as a result of a pivot shift mechanism of injury and are diagnostic for an ACL rupture. The most common associated injury in an acute ACL rupture is a lateral meniscus tear. Medial meniscus tears are more common in chronic ACL injuries. PCL rupture and PLC injury are all associated injuries seen in acute ACL rupture; however, these are much less common than meniscal tears. The team physician has a role in encouraging, but

 

not demanding, the athlete to report the injury andtreatment to the scholarship school’s coaching staff. What the athlete decides to do is his decision; the physician would be violating the athlete’s HIPAA rights as well as their confidence by reporting it directly to the scholarship school. Clearly the physician should not discourage the athlete from reporting the injury. The athlete is 18 years old and, as such, the physician would need the athlete’s permission to discuss any medical issues with the family in keeping with HIPAA.

The athlete’s exam demonstrates incompetence of both bundles of the ACL as demonstrated by the loss of stability with anterior translation of the tibia (Lachman test) as well as with rotation (pivot shift). The external rotation stress with the knee in 30 degrees of flexion tests the competence of the posterolateral corner while rotation at 90 degrees of flexion tests the PCL. Since the athlete’s knee is stable to posterior drawer testing demonstrating an intact PCL and the external rotation at 30 degrees is equivalent to that at 90 degrees, the posterolateral corner in intact.

The ACL has two separate and distinct bundles, the AM and PL. Each bundle takes on tension at varying degrees of knee flexion and therefore each bundle is thought to have a varying contribution to the stability of the knee. The AM bundle takes on tension with the knee in flexion and the PL bundle is tight in extension. Neither bundle is isometric during knee range of motion. Both bundles have contributions to rotational stability of the knee throughout the range of motion.

The success of traditional trans-tibial single-bundle ACL reconstruction has recently been called into question given the demonstration of persistent rotational instability following reconstruction. The persistence of rotational instability in trans-tibial single bundle ACL reconstruction has been attributed to the location of the graft in a vertically malpositioned femoral tunnel. The goal of double-bundle ACL reconstruction is to more accurately reproduce the native ACL and provide grafts that contribute to anteroposterior stability as well as rotational stability by placing the grafts in more anatomic locations not central in the knee axis. There is an increased cost and surgical time associated with double-bundle reconstruction.

The use of functional braces following ACL reconstruction is a surgeon’s preference because there is no difference in retear rate with or without a brace. Some authors recommend brace use for one to two years following ACL reconstruction for all athletic activities, but this is not supported by the literature. No literature exists showing a higher rate of reinjury with a functional brace and off-the shelf and custom braces have been found to be equivalent leading those who advocate for braces to recommend off-theshelf braces given their significantly lower cost.

 

 

 

Question 96-Figures 96a and 96b are the MRI scans of a 57-year-old man who dislocated his left shoulder after a fall while playing tennis. On examination he had full passive shoulder range of motion, but he was unable to actively elevate his injured shoulder. Sensation was intact to light touch over the lateral shoulder. What is the most likely etiology of his shoulder weakness?

 

  1. Axillary nerve injury

  2. Cervical radiculopathy involving the C6 nerve root

  3. Massive rotator cuff tear with loss of the transverse force couple

  4. Long head of the biceps tendon rupture with loss of superior stabilizing effect

 

DISCUSSION --This patient has a massive rotator cuff tear resulting in disruption of the transverse force couple between the subscapularis anteriorly and the infraspinatus and teres minor posteriorly. These muscles provide dynamic shoulder stability throughout active elevation, and loss of the force couple produces a pathologic increase in translation of the humeral head and decreased active abduction. Active shoulder elevation less than 90 degrees in the presence of full passive motion is termed pseudoparalysis. The most common neurologic deficit after shoulder dislocation is isolated injury to the axillary nerve. This patient’s sensory examination suggests that the axillary nerve is intact. Cervical radiculopathy is less common after shoulder dislocation but has been reported.

Conflicting evidence exists regarding the contribution of the long head of the biceps tendon to glenohumeral stability. One study reported minimal electromyographic activity in the biceps during 10 basic shoulder motions. PREFERRED RESPONSE: 3

 

CLINICAL SITUATION FOR QUESTIONS 97 AND 98

A 14-year-old gymnast missed her dismount off of the uneven bars, hit the mat face first, and lost consciousness for about 15 seconds. She was dazed and confused for several minutes. She does not complain of pain, numbness, or weakness and she is moving all extremities without deficit.

 

Question 97--The athlete and coach want to go back to competition that day. How should they be advised?

  1. Concussion precludes same-day return to play.

  2. Order an urgent MRI scan; if findings are normal, she can return to competition.

  3. Order neurocognitive testing; if findings are normal, she can return to competition.

  4. If she is symptom-free after a 15-minute exertional test, she may return to competition.

 

PREFERRED RESPONSE: 1

Question 98-Thirty minutes later the gymnast is experiencing headache and difficulty concentrating. If her symptoms persist 1 week later, the next treatment step should be

  1. a functional MRI scan.

  2. serial neurocognitive testing.

  3. no return to play that season.

  4. cognitive, cranial nerve, and balance testing after a period of moderate-to-intense exercise.

 

PREFERRED RESPONSE: 2

DISCUSSION FOR QUESTIONS 97 AND 98

The National Collegiate Athletic Association’s (NCAA) 2011 revised health and safety guidelines regarding concussion management (available at www.ncaa.org) recommend no return to play on the same day of an injury. In particular, athletes sustaining a concussion should not return to play the same day as their injury. Before resuming exercise, athletes must be asymptomatic or returned to baseline symptoms at rest and have no symptoms with cognitive effort. They must be off of medications that could mask or alter concussion symptoms. Neurocognitive testing can be a helpful tool in determining brain function even after all symptoms of concussion have resolved. With a comparison baseline test, this evaluation, in conjunction with a physician’s examination, may reduce risk for second impact syndrome. The athlete’s clinical neurological examination findings (cognitive, cranial nerve, and balance testing) must return to baseline before resuming exercise. Research has shown that among youth athletes it may take longer for tested functions to return to baseline

 

(compared to the recovery rate in adult athletes). Brain MRI scan has no role in evaluating athletes for return to play in this situation.

 

Question 99-A 24-year-old collegiate pitcher has had increasing pain over his medial elbow for 3 months. He has point tenderness over his medial epicondyle and reproduction of his symptoms with a valgus stress test. What phase of the throwing cycle most likely will reproduce his symptoms?

  1. Early cocking

  2. Late cocking

  3. Acceleration

  4. Deceleration

 

DISCUSSION --This patient is experiencing soreness over his medial (ulnar) collateral ligament. Valgus overload is likely to reproduce his symptoms and is most pronounced during the late cocking phase of the throwing cycle.In wind up, very little elbow torque is required. In early cocking, the arm is getting loaded and maximum valgus is not yet achieved at the elbow. In acceleration and deceleration, more force is generated at the level of the shoulder joint.

 

PREFERRED RESPONSE: 2

 

 

 

Question 100--Figure 100 is the MRI scan of a 52-year-old runner who has right knee pain that has been occurring 10 minutes into her run for 2 months. On examination, she has tenderness over the lateral epicondyle. Her Ober test result is positive. What is the most appropriate initial treatment?

 

  1. Iliotibial band bursectomy

  2. Z-lengthening of the iliotibial band

  3. Partial excision of the iliotibial band

  4. Home stretching program and cross training

 

DISCUSSION -Iliotibial band syndrome (ITBS) is a common cause of lateral knee pain in runners. Potential etiologies for the pain include repetitive friction, compression, and bursal inflammation. An Ober test is used to assess iliotibial band tightness. With the patient lying on the unaffected side, the affected leg is abducted and extended. The test result is positive if the examiner is unable to adduct the leg from this position. An MRI scan can be helpful in making the diagnosis, but a negative MRI scan does not ruleout ITBS. Studies have reported increased signal intensity on T2-weighted images deep to the iliotibial band adjacent to the lateral epicondyle, with thickening of the iliotibial band.

Nonsurgical treatment is most appropriate initially and involves activity modification, ice, anti-inflammatory medications, and stretching. Corticosteroid injection to the iliotibial bursa is also an option to treat acute pain. After the initial inflammation improves, a strengthening program is started. Multiple surgical procedures have been described for recalcitrant cases, including iliotibial band excision, Z-lengthening, and iliotibial band bursectomy.

 

PREFERRED RESPONSE: 4